similar to: quadratically constrained quadratic programming

Displaying 20 results from an estimated 2000 matches similar to: "quadratically constrained quadratic programming"

2012 Apr 17
1
What functions are available for Quadratically Constrained Quadratic Programming in R?
Hi all, Could anybody please point me to the solver function in R on QCQP? The quadprog package seems to be only able to handle the linear constraints... Thank you! [[alternative HTML version deleted]]
2007 Jul 11
0
Some questions about quadratic programming (QP)
Dear R Users , As a beginner in QP, I'm trying to solve a Support Vector Machine problem by a QP. In particulare I am using the quadprog package. My questions are here: 1- In the document for the package (The quadprog Package), the inequality constraint is mentioned with >= , however in a standard QP, this usaully is written with <= . This constraint should be multiplied by a
2006 Jun 06
1
Problems using quadprog for solving quadratic programming problem
Hi, I'm using the package quadprog to solve the following quadratic programming problem. I want to minimize the function (b_1-b_2)^2+(b_3-b_4)^2 by the following constraints b_i, i=1,...,4: b_1+b_3=1 b_2+b_4=1 0.1<=b_1<=0.2 0.2<=b_2<=0.4 0.8<=b_3<=0.9 0.6<=b_4<=0.8 In my opinion the solution should be b_1=b_2=0.2 und b_3=b_4=0.8. Unfortunately R doesn't find
2010 Dec 04
1
Quadratic programming with semi-definite matrix
Hello. I'm trying to solve a quadratic programming problem of the form min ||Hx - y||^2 s.t. x >= 0 and x <= t using solve.QP in the quadprog package but I'm having problems with Dmat not being positive definite, which is kinda okay since I expect it to be numerically semi-definite in most cases. As far as I'm aware the problem arises because the Goldfarb and Idnani method first
2008 Feb 15
2
Quadratic Programming
Hi, I am using solve.QP (from quadprog) to solve a standard quadratic programming problem: min_w -0.5*w'Qw st ... I would like solve.QP to do two things: 1) to start the optimization from a user-supplied initial condition; i.e., from a vector w_0 that satisfies the constraints, and 2) to return the values of the lagrange multiplieres associated with the constraints. I did not find an obvious
2010 May 18
1
Maximization of quadratic forms
Dear R Help, I am trying to fit a nonlinear model for a mean function $\mu(Data_i, \beta)$ for a fixed covariance matrix where $\beta$ and $\mu$ are low- dimensional. More specifically, for fixed variance-covariance matrices $\Sigma_{z=0}$ and $\Sigma_{z=1}$ (according to a binary covariate $Z $), I am trying to minimize: $\sum_{i=1^n} (Y_i-\mu_(Data_i,\beta))' \Sigma_{z=z_i}^{-1} (Y_i-
2007 Dec 05
1
Quadratic programming
Hi, I'm quite new at R and I haven't found the answer to my question anywhere on the net, so either it is trivial or not documented. So, bare with be. I am using the quadprog package and its solve.QP routine to solve and quadratic programming problem with inconsistent constraints, which obviously doesn't work since the constraint matrix doesn't have full rank. A way to solve this
2007 Dec 22
1
using solve.qp without a quadratic term
I was playing around with a simple example using solve.qp ( function is in the quadprog package ) and the code is below. ( I'm not even sure there if there is a reasonable solution because I made the problem up ). But, when I try to use solve.QP to solve it, I get the error that D in the quadratic function is not positive definite. This is because Dmat is zero because I don't have a
2007 Dec 14
1
Quadratic Programming
Hi all! I have a little question concerning quadprog. To make it simple I'll start by stating the problem: I want to minimize h(d,delta)=0.5d^T B d +nabla(f(x))^T d +rho*delta^2 With respect to d\in R^n and delta \in R. I obviously have constraints (depending on both d and delta). Solve.QP does give me a good result for d but I cannot obtain anything for delta. Simce dim(Dmat)=n and
2005 Mar 30
0
xen problem
hi, lately i try to download xen from bitkeeper - bk://xen.bkbits.net/xen-2.0.bk (ought to be version 2.0); but encounter a probelm when build from source. it has error as follow. the env is cygwin, gcc 3.3.3 and make 3.80 how to slove it? i appreciate any suggestion. sinrecely. jason ===============ERROR BEG ============ ac_timer.c:44: warning: alignment of `ac_timers'' is greater than
2004 Sep 01
0
not positive definite D matrix in quadprog
Hello to everybody, I have a quadratic programming problem that I am trying to solve by various methods. One of them is to use the quadprog package in R. When I check positive definiteness of the D matrix, I get that one of the eigenvalues is negative of order 10^(-8). All the others are positive. When I set this particular eigenvalue to 0.0 and I recheck the eigenvalues in R, the last
2006 Aug 09
1
minimization a quadratic form with some coef fixed and some constrained
Hello, all, I had problems with an extension to a classic optimization problem. The target is to minimize a quadratic form a'Ma with respect to vector b, where vector a=(b',-1)', i.e., a is the expand of b, and M is a symmetric matrix (positive definite if needed). One more constrain on b is b'b=1. I want to solve b given M. I tried but it seems impossible to find an analytic
2001 Nov 20
0
Summary: non-negative least squares
Thank you Brian Ripley, Gardar Johannesson, and Marcel Wolbers for your prompt and friendly help! I will share any further learnings as I move through these suggestions. -Bob Abugov Brian Ripley wrote: I just use optim() on the sum of squares with non-negativity constraints. That did not exist in 1999. Gardar Johannesson wrote: You can always just use the quadratic programing library in R
2011 Jan 03
1
Greetings. I have a question with mixed beta regression model in nlme.
*Dear R-help: My name is Rodrigo and I have a question with nlme package in R to fit a mixed beta regression model. The details of the model are: Suppose that:* *j in {1, ..., J}* *(level 1)* *i in {1, ..., n_j}* *(level 2)* *y_{ij} ~ Beta(mu_{ij} * phi_{ij}; (1 - mu_{ij}) * phi_{ij}) y_{ij} = mu_{ij} + w_{ij} * *with* *logit(mu_{ij}) = Beta_{0i} + Beta_{1i} * x1_{ij} + b2 * x2_{ij}
2011 Jan 03
0
Greetings. I have a question with mixed beta regression model in nlme (corrected version).
*Dear R-help: My name is Rodrigo and I have a question with nlme package in R to fit a mixed beta regression model. I'm so sorry. In the last email, I forgot to say that W is also a unknown parameter in the mixed beta regression model. In any case, here I send you the correct formulation. ** Suppose that:* *j in {1, ..., J}* *(level 1)* *i in {1, ..., n_j}* *(level 2)* *y_{ij} ~
2010 Feb 23
2
significance of coefficients in Constrained regression
I am fittting a linner regression with constrained parameters, saying, all parameters are non-negative and sum up to 1. I have searched historical R-help and found that this can be done by solve.QP from the quadprog package. I need to assess the significance of the coefficient estimates, but there is no standard error of the coefficient estimates in the output. So I can not compute the p-value.
2008 Aug 29
3
extract variance components
HI, I would like to extract the variance components estimation in lme function like a.fit<-lme(distance~age, data=aaa, random=~day/subject) There should be three variances \sigma_day, \sigma_{day %in% subject } and \sigma_e. I can extract the \sigma_e using something like a.fit$var. However, I cannot manage to extract the first two variance components. I can only see the results in
2002 May 28
1
constrained regression
I want to do a linear regression where the coefficients obey two linear constraints, and also are all non-negative. What is the best way to do this? Computational speed is a consideration as I must do it many times. When this question was asked previously on the list, quadprog was suggested - is this the best solution? (I may have missed something obvious in the documentation, but I have
2008 Apr 10
2
QP.solve, QPmat, constraint matrix, and positive definite
hello all, i'm trying to use QPmat, from the popbio package. it appears to be based on solve.QP and is intended for making a population projection matrix. QPmat asks for: nout, A time series of population vectors and C, C constraint matrix, (with two more vectors, b and nonzero). i believe the relevant code from QPmat is: function (nout, C, b, nonzero) { if (!"quadprog" %in%
2002 Aug 21
4
Quadratic optimization problem
I hope that someone can help me with the following question: I would like to solve the Markowitz optimization problem WITH short-sale constraints. Maybe a procedure to solve a quadratic optimization problem with convex constraints and positive variables is already implemented in R? Thank you very much, edg -.-.-.-.-.-.-.-.-.-.-.-.-.-.-.-.-.-.-.-.-.-.-.-.-.-.-.-.-.-.-.-.-.-.-.-.-.-.-.- r-help